K
Khách

Hãy nhập câu hỏi của bạn vào đây, nếu là tài khoản VIP, bạn sẽ được ưu tiên trả lời.

4 tháng 2

 Giả sử \(r+\sqrt{a}\) là một số hữu tỉ. Đặt \(r+\sqrt{a}=\dfrac{p}{q}\) với \(p,q\inℤ\)\(q\ne0\) và \(\left(p,q\right)=1\)

 \(\Leftrightarrow r=\dfrac{p}{q}-\sqrt{a}\)

 Vì \(r^3-2ar+1=0\)

 \(\Leftrightarrow\left(\dfrac{p}{q}-\sqrt{a}\right)^3-2a.\left(\dfrac{p}{q}-\sqrt{a}\right)+1=0\)

 \(\Leftrightarrow\dfrac{p^3}{q^3}-\dfrac{3p^2\sqrt{a}}{q^2}+\dfrac{3ap}{q}-a\sqrt{a}-\dfrac{2ap}{q}+2a\sqrt{a}+1=0\)

\(\Leftrightarrow\dfrac{p^3}{q^3}-\dfrac{3p^2\sqrt{a}}{q^2}+\dfrac{ap}{q}+a\sqrt{a}+1=0\)

 \(\Leftrightarrow\dfrac{p^3+apq^2+q^3}{q^3}+\left(\dfrac{aq^2-3p^2}{q^2}\right)\sqrt{a}=0\)

 Vì \(p,q,a\inℤ\) nên \(\dfrac{p^3+apq^2+q^3}{q^3}\) và \(\dfrac{aq^2-3p^2}{q^2}\) là các số hữu tỉ. Hơn thế nữa, 0 cũng là một số hữu tỉ, trong khi đó \(\sqrt{a}\) lại là số vô tỉ (vì \(a\) là số nguyên dương không chính phương) nên \(\dfrac{aq^2-3p^2}{q^2}=0\)

 \(\Leftrightarrow aq^2=3p^2\) 

 Nếu \(3⋮a\Rightarrow a\in\left\{1,3\right\}\). Với \(a=1\) thì \(q^2=3p^2\) \(\Rightarrow q⋮3\) \(\Rightarrow q=3k\left(k\inℤ\right)\) 

 \(\Rightarrow9k^2=3p^2\) \(\Rightarrow p^2=3k^2\) \(\Rightarrow p⋮3\). Từ đây ta có \(p,q⋮3\) , mẫu thuẫn với điều kiện \(\left(p,q\right)=1\)

  Với \(a=3\) thì \(q^2=p^2\) \(\Leftrightarrow q=\pm p\) \(\Leftrightarrow r+\sqrt{3}=\pm1\) hay \(r=-\sqrt{3}\pm1\)

 Trong trường hợp này, ta thấy \(r^3-2ar+1=\left(-\sqrt{3}\pm1\right)^3-6\left(-\sqrt{3}\pm1\right)+1\ne0\) nên \(a=3\) không thỏa mãn.

 Vậy \(3⋮̸a\) \(\Rightarrow p⋮a\) \(\Rightarrow p=al\left(l\inℤ\right)\)

  \(\Rightarrow aq^2=3\left(al\right)^2\) 

  \(\Leftrightarrow q^2=3al^2\) 

 \(\Rightarrow q⋮a\)

 Vậy \(p,q⋮a\). Do \(a>1\) nên từ đây, ta thấy mâu thuẫn với điều kiện \(\left(p,q\right)=1\)

 Do đó, điều giả sử là sai \(\Rightarrow r+\sqrt{a}\in I\)

 

 

4 tháng 2

 Ở chỗ cuối mình xét thiếu. Từ pt \(aq^2=3p^2\), nếu \(a=3t\) mà \(t\) không phải là SCP thì có \(tq^2=p^2\) \(\Rightarrow p⋮t\) \(\Rightarrow p=tu\) \(\Rightarrow tq^2=t^2u^2\) \(\Rightarrow q^2=tu^2\) \(\Rightarrow q⋮t\) \(\Rightarrow p,q⋮t\), mâu thuẫn.

 Còn nếu \(a=3c^2\left(c\ge2\right)\) thì \(p^2=c^2q^2\) \(\Leftrightarrow p=\pm cq\) \(\Leftrightarrow\dfrac{p}{q}=\pm c\) 

 Lại có \(r=\dfrac{p}{q}-\sqrt{a}=-c\sqrt{3}\pm c\)

 Nếu \(r=-c\sqrt{3}+c\) thì \(r^3-2ar+1=\left(-c\sqrt{3}+c\right)^3-6\left(-c\sqrt{3}+c\right)+1\) \(=4c^3+1>0\) với \(c\ge2\), vô lí.

 Nếu \(r=-c\sqrt{3}-c\) thì

\(r^3-2ar+1=-4c^3+1< 0\) với \(c\ge2\), vô lí.

 Giờ ta mới xét đủ trường hợp để chứng minh giả sử sai.

11 tháng 12 2022

Bài 2: 

Ta có: 2a2+2b2=(a2+2ab+b2)+(a2-2ab+b2)

                        =(a+b)2+(a-b)2 là tổng 2 số chính phương

⇒2a2+2b2 là tổng của 2 số chính phương(đpcm)

13 tháng 1 2022

32+1123+ \(x = {-b \pm \sqrt{b^2-4ac} \over 2a}gfdrrffhjxxojmu09\)

11 tháng 3 2019

Bài 1. x^2 \(\equiv\)8 (mod 0,1). (cmdd)

T tự: y^2 \(\equiv\)8 (mod 0,1)

=> x^2+y^2 \(\equiv\)8 (mod 0,1,2)

Mà 8z+6 \(\equiv\)8 (mod 6)

=> đpcm

25 tháng 8 2015

3. a) Coi A = ab+1
A = 111...11(n chữ số 1) .10+ 5 .111...11(n chữ số 1) + 1
 \(A= \frac {10^n - 1} {9} + 5 \frac { 10^n -1} {9}+1 \)

\(A= \frac {10^2n - 10^n + 5.10^n -5 + 9} {9}\)

\(A =\frac {10^{2n} + 4.10^n + 4} {9}\)

\(A =\frac {(10^n + 2)^2} {3^2}\)

\(A=(\frac{10^n+2} {3}) ^2\)
Vậy A là số chính phương (vì 10n+2 chia hết cho 3)

 

b)Ta thấy 16 = 1.15 + 1
               1156 = 11.105 + 1
               111556 = 111.1005 + 1
...            111...1555...56(n chữ số 1,n-1 chữ số 5) = 111...1(n chữ số 1).100...05(n-1 chữ số 0) +1 (phần a)
               Vẫy các số hạng trong dãy trên đều là số chính phương

11 tháng 7 2015

3a)(dấu * là nhân nhé)

Có ab+1

=11...1*100...05+1

=11...1*(33...35(n-1 chữ số 3)*3)+1

=33...3*33...35+1

=33...3*(33...34+1)+1

=33...3*33...34+(33...3+1)

=33...3*33...34+33...34(n-1 chữ số 3)

=33...34*(33...3+1)

=33...34*33...34(n-1 chữ số 3)

=(33...34)^2 là số chính phương

27 tháng 5 2021

thật ra nó là lớp 7 đấy nhưng mình nghĩ lớp 8 mới giỏi mói giải đc

 

27 tháng 5 2021

Giả sử \(a^2+1\) và \(b^2+1\) cùng chia hết cho số nguyên tố p

\(\Rightarrow a^2-b^2⋮p\)

\(\Rightarrow\left(a-b\right)\left(a+b\right)⋮p\Rightarrow\left[{}\begin{matrix}a-b⋮p\\a+b⋮p\end{matrix}\right.\).

+) Nếu \(a-b⋮p\) thì ta có \(\left(a^2+1\right)\left(b^2+1\right)-\left(a-b\right)^2⋮p\Rightarrow\left(ab+1\right)^2⋮p\Rightarrow ab+1⋮p\) (vô lí do (a - b, ab + 1) = 1)

+) Nếu \(a+b⋮p\) thì tương tự ta có \(ab-1⋮p\). (vô lí)

Do đó \(\left(a^2+1,b^2+1\right)=1\).

Giả sử \(\left(a+b\right)^2+\left(ab-1\right)^2=c^2\) với \(c\in\mathbb{N*}\)

Khi đó ta có \(\left(a^2+1\right)\left(b^2+1\right)=c^2\).

Mà \(\left(a^2+1,b^2+1\right)=1\) nên theo bổ đề về số chính phương, ta có \(a^2+1\) và \(b^2+1\) là các số chính phương.

Đặt \(a^2+1=d^2(d\in\mathbb{N*})\Rightarrow (d-a)(d+a)=1\Rightarrow d=1;a=0\), vô lí.

Vậy ....

NM
13 tháng 1 2022

a. để phương trình nhận x=3 là nghiệm ta có 

\(a\left(3+2\right)-a^2-2=0\Leftrightarrow a^2-5a+2=0\Leftrightarrow a=\frac{5\pm\sqrt{17}}{2}\)

b. Để phương trình có duy nhất 1 nghiệm âm ta có : 

\(\hept{\begin{cases}a\ne0\\x=\frac{a^2-2a+2}{a}< 0\end{cases}\Leftrightarrow a< 0}\) do \(a^2-2a+2>0\forall a\)

c. Để phương trình đã cho vô nghiệm thì a=0

d. Phương trình đã cho không thể có vô số nghiệm thực.